This is a weaken question - and in particular a "most weakens" question, meaning we need to take a moment to eyeball each answer. But first, of course, we need to figure out the core of this argument. Let's take a look.
The conclusion is the first sentence - that people who take Vitamin C tend to be healthier than those who don't - a conclusion reached on somewhat narrow (only heart disease) grounds. Another problem is that the conclusion talks about vitamin C, but we only know about high doses not regular doses.
(A) is irrelevant - we don't care about how the vitamin C is taken. This doesn't relate to anything in the conclusion or in the premises. Eliminate this on the first go round.
(B) might at first seem tempting, but all it is doing is limiting how good Vitamin C can possibly be. But we are interested in the more fundamental claim that it is good for you/creates better health. We are not interested in comparing it to other vitamins/nutrients/etc. Eliminate this on the first go round.
(C) is no good either. We don't care about Vitamin E - all this does is tell us that the effect we are arguing about here can be amplified, but we are trying to disprove the effect concluded by the author in the first place! Eliminate this on the first go round.
(D) is good in referring specifically to *high doses* of Vitamin C, since that is what we had in our conclusion. This weakens the conclusion because it uses the narrow scope of the evidence (only about heart disease) as an opportunity to weaken the argument instead by pointing to contrary (but not contradictory evidence) about negative health effects in another domain. This is a good option, so we'll hold onto it.
(E) strengthens the argument. Eliminate this.
We're only left with (D) - this is the only answer choice that weakens the argument at all.
This is probably a difficult problem because it looks like a correlation/causation problem, but it really isn't. It's good to have your eyes open for such an argument situation, but also be careful about not seeing it where it isn't.
Any questions about this? I'm happy to help
#officialexplanation